Themenbereiche Themenbereiche Profile Hilfe/Anleitungen Help    
Recent Posts Last 1|3|7 Days Suche Suche Tree Tree View  

Bitte Checken: Beweis Fermat für n=3!...

ZahlReich - Mathematik Hausaufgabenhilfe » ---- Archiv: Universitäts-Niveau » Zahlentheorie » Bitte Checken: Beweis Fermat für n=3! « Zurück Vor »

Autor Beitrag
Seitenanfangvoriger Beitragnächster BeitragSeitenende Link zu diesem Beitrag

Reti
Suche alle Beiträge dieser Person in dieser Hauptrubrik
Veröffentlicht am Samstag, den 15. September, 2001 - 18:14:   Beitrag drucken

Hi,
könnte Ihr mir den folgenden Beweis für Fermat's
letzen Satz für n=3 durchchecken. Mir kommt
er zwar reichlich logisch, aber sehr (zu?) einfach
vor, dennoch bitte kontrolliert das mal:

Man nimmt an a^3 + b^3 =c^3 existiert und setzt
a=(2*n+1)
b=(2*x+1)
nun ist c^3= 8x^3 + 12x^2 + 6x + ((2*n+1)+1)
der Term rechts des Gleichheitszeichens kann
nun als Polynomfunktion f(x) mit Lösungsvariable x
und Parameter n aufgefasst werden.
Falls c natürliche Zahl ist (c^3 ist also die dritte Potenz einer natürlichen Zahl), so muss
sich das Polynom f(x)=8x^3 + 12x^2 + 6x + ((2*n+1)+1) in der Form T^3 (T ist ein Linearfaktor) darstellen lassen.
Oder mit anderen Worten: f(x) muss eine dreifache
natürliche Nullstelle haben.
Daher folgt, dass f(x), f'(x) und f''(x) dieselben
Lösungen haben, also

f(x)= 8x^3 + 12x^2 + 6x + ((2*n+1)+1)
f'(x)= 24x^2 + 24x + 6
f''(x)= 48x + 24

f''(x)=0 <=> x=-1/2

diese Lösung müsste aber natürlich sein, damit
f(x) eine dreifache natürliche Nullstelle hätte
und folglich c^3 eine natürlich Zahl wäre.
Hier ergibt sich ein Wiederspruch, womit gezeit
ist, dass a^3 + b^3 =c^3 mit a,b,c Element N nicht
existieren kann.

Reti
Seitenanfangvoriger Beitragnächster BeitragSeitenende Link zu diesem Beitrag

Hans (Birdsong)
Suche alle Beiträge dieser Person in dieser Hauptrubrik
Veröffentlicht am Samstag, den 15. September, 2001 - 19:01:   Beitrag drucken

Reti :

Wenn ich dich richtig verstehe, so behauptest
du:

Wenn ein Polynom 3. Grades f(x) fŸr ein natŸrliches Argument x = m die 3. Potenz einer
nat. Zahl c ist, also f(m) = c^3, so ist f(x)
selbst die dritte Potenz eines linearen Polynoms,
f(x) = (ax+b)^3.
Das ist natŸrlich falsch, beliebig viele
Gegenbeispiele kannst du dir leicht selbst
konstruieren.

mfG

Hans
Seitenanfangvoriger Beitragnächster BeitragSeitenende Link zu diesem Beitrag

Ex
Suche alle Beiträge dieser Person in dieser Hauptrubrik
Veröffentlicht am Samstag, den 15. September, 2001 - 23:55:   Beitrag drucken

Hallo Reti, ich erkenne nicht, wie du auf
c^3= 8x^3 + 12x^2 + 6x + ((2*n+1)+1)
kommst.

Wenn ich a^3=(2*n+1)^3 ausrechne, ist das bei mir gleich
8*n^3 + 3*4*n^2 + 3*2*n + 1 = 8*n^3+12*n^2+6*n+1,
und damit ergibt sich c^3 zu

c^3 = 8x^3 + 12x^2 + 6x + 8*n^3+12*n^2+6*n+2

Das n soll gleich 3 sein?
selbst mit a=2*3+1 => a^3=343 erhalte ich nicht deine Formel.
?
Seitenanfangvoriger Beitragnächster BeitragSeitenende Link zu diesem Beitrag

Reti
Suche alle Beiträge dieser Person in dieser Hauptrubrik
Veröffentlicht am Sonntag, den 16. September, 2001 - 09:24:   Beitrag drucken

@Ex:
Natürlich sollte der Term (2*n+1) in die
dritte Potenz erhoben sein.
f(x)=8x^3 + 12x^2 + 6*x + ((2n+1)^3+1)

@Hans:

Weshalb ist das Falsch?
Wenn doch c^3=f(x) ist und f(x) ein Polynom dritten Grades ist, so ist doch c = 3. Wurzel aus
f(x). Damit diese eine natürliche Zahl ist, muss
f(x) als Term (ax+b)^3 mit x Element N darstellbar sein. Dh. f(x) muss notwendigerweise
eine dreifache natürliche Nullstelle haben.
Bitte um Aufklärung, sehe den Fehler nicht, viellicht. Evtl. vor lauter Bäume den Wald übersehen.....

Reti
Seitenanfangvoriger Beitragnächster BeitragSeitenende Link zu diesem Beitrag

Hans (Birdsong)
Suche alle Beiträge dieser Person in dieser Hauptrubrik
Veröffentlicht am Sonntag, den 16. September, 2001 - 15:51:   Beitrag drucken

Reti,

Wenn f(m) = c^3 mit m,c in N, wenn also f(x) fŸr den s p e z i e l l e n Wert x=m als Funktionswert eine 3.Potenz ergibt, so heisst das nicht, dass f(x) = (ax+b)^3 als Polynomidentitaet, also fŸr a l l e x in R erfŸllt sein muss.

Beispiel: f(x) = 2 x^3 + x^2 + 3 x + 1,
f(2) = 27 = 3^3, aber offenbar ist f(x) keine
3. Potenz.

mfG

Hans
Seitenanfangvoriger Beitragnächster BeitragSeitenende Link zu diesem Beitrag

N.
Suche alle Beiträge dieser Person in dieser Hauptrubrik
Veröffentlicht am Sonntag, den 16. September, 2001 - 17:06:   Beitrag drucken

Hallo Reti,

vieleicht hilft dir folgender Link weiter...

Beweis des Satzes von Fermat für n=3

Gruß N.

Beitrag verfassen
Das Senden ist in diesem Themengebiet nicht unterstützt. Kontaktieren Sie den Diskussions-Moderator für weitere Informationen.

ad

Administration Administration Abmelden Abmelden   Previous Page Previous Page Next Page Next Page